Đến nội dung

Hình ảnh

cho a,b,c dương chứng minh bất đẳng thức sau đúng

- - - - -

  • Please log in to reply
Chủ đề này có 1 trả lời

#1
trandinhhuy

trandinhhuy

    Binh nhất

  • Thành viên
  • 35 Bài viết
1)CMR với tam giác tùy ý có độ dài cạnh a,b,c thì bđt sau luôn đũng
(2a²+2b²-c²)(2b²+2c²-a²)(2c²+2a²-b²)<=(2a²+bc)(2b²+ca)(2c²+ab) dấu = xảy ra khi nào ?
2)cho a,b,c>0 CMR (a²+b²)/(a+b)+(b²+c²)/(b+c)+(c²+a²)/(c+a)<=3(a²+b²+c²)/a+b+c
3)cho a,b,c>0 a³/(a²+ab+b²)+b³/(b²+bc+c²)+c³/(c²+ca+a²)>= (a+b+c)/3
4)cho 0<a,b,c,d<=1 CMR a/(bcd+1)+b/(cda+1)+c/(dab+1)+d/(abc+1)<=3

#2
Rias Gremory

Rias Gremory

    Del Name

  • Thành viên
  • 1384 Bài viết

1)CMR với tam giác tùy ý có độ dài cạnh a,b,c thì bđt sau luôn đũng
(2a²+2b²-c²)(2b²+2c²-a²)(2c²+2a²-b²)<=(2a²+bc)(2b²+ca)(2c²+ab) dấu = xảy ra khi nào ?
2)cho a,b,c>0 CMR (a²+b²)/(a+b)+(b²+c²)/(b+c)+(c²+a²)/(c+a)<=3(a²+b²+c²)/a+b+c
3)cho a,b,c>0 a³/(a²+ab+b²)+b³/(b²+bc+c²)+c³/(c²+ca+a²)>= (a+b+c)/3
4)cho 0<a,b,c,d<=1 CMR a/(bcd+1)+b/(cda+1)+c/(dab+1)+d/(abc+1)<=3

Sửa lại đề cho dễ nhìn, mong bạn lần sau Gõ LATEX .

$1$ , CMR với tam giác tùy ý có độ dài cạnh $a,b,c$ thì BĐT sau luôn đúng :

$(2a^{2}+2b^{2}-c^{2})(2b^{2}+2c^{2}-a^{2})(2c^{2}+2a^{2}-b^{2})\leq (2a^{2}+bc)(2b^{2}+ca)(2c^{2}+ab)$ 

Dấu bằng xảy ra khi nào ??

$2$, Cho $a,b,c> 0$ . Chứng minh : 

$\frac{a^{2}+b^{2}}{a+b}+\frac{b^{2}+c^{2}}{b+c}+\frac{c^{2}+a^{2}}{c+a}\leq \frac{3(a^{2}+b^{2}+c^{2})}{a+b+c}$

$3$, Cho $a,b,c> 0$. Chứng minh:

$\frac{a^{3}}{a^{2}+ab+b^{2}}+\frac{b^{3}}{b^{2}+bc+c^{2}}+\frac{c^{3}}{c^{2}+ca+a^{2}}\geq \frac{a+b+c}{3}$

$4$, Cho $0< a,b,c,d\leq 1$. Chứng minh

$\frac{a}{bcd+1}+\frac{b}{cda+1}+\frac{c}{dab+1}+\frac{d}{abc+1}\leq 3$






1 người đang xem chủ đề

0 thành viên, 1 khách, 0 thành viên ẩn danh